Proving Contraction Constants: Mean Value Theorem Help | Homework Example

Click For Summary
To prove that the composite function h2 ° h1 is a contraction on set B with contraction constants δ1 and δ2, the Mean Value Theorem and chain rule are essential tools. The inequality |h2(h1(a)) - h2(h1(b))| can be established using the contraction property of h2, leading to |h2(h1(a)) - h2(h1(b))| ≤ δ2|h1(a) - h1(b)|. This process can be repeated by applying the contraction property of h1, resulting in |h2(h1(a)) - h2(h1(b))| ≤ δ2δ1|a - b|. The discussion highlights the importance of understanding the properties of contractions and how they interact in composite functions. Ultimately, the conclusion is that the composite function is indeed a contraction with a contraction constant of δ2δ1.
Easty
Messages
19
Reaction score
0

Homework Statement



If h1 and h2 are contractions on a set B with contraction constants δ1 and δ2 prove that the composite function h2 ° h 1 is also a contraction on B and find a contraction constant.


Homework Equations




|f(a) - f(b)| ≤ δ |a-b|

f '(c) = (f(a)-f(b))/(a-b)


(g°f) '(c) = g '(f(c))x f '(c)


The Attempt at a Solution



So far I'm pretty sure i have to use the mean value theorem and the chain rule. using the mean value on the composite function i get :

|h2(h1(a)) - h2(h1(b))| = |h2 °h1 )' (c)| |a-b|

i get stuck here, i think i should now use the chain rule for the derivaitve term out the front of the equality to somehow make an inequality. am i on the right track?
 
Last edited:
Physics news on Phys.org


I don't see why you should use derivatives at all. If h(x)= h_2(h_1(x)) then |h(a)- h(b)|= |h_2(h_1(a))- h_2(h_1(b))|\le \delta_2|h_1(a)- h_1(b)| and repeat.
 


HallsofIvy said:
I don't see why you should use derivatives at all. If h(x)= h_2(h_1(x)) then |h(a)- h(b)|= |h_2(h_1(a))- h_2(h_1(b))|\le \delta_2|h_1(a)- h_1(b)| and repeat.


but then arent you assumeing that h(x)= h_2(h_1(x)) is a contration?
 
Question: A clock's minute hand has length 4 and its hour hand has length 3. What is the distance between the tips at the moment when it is increasing most rapidly?(Putnam Exam Question) Answer: Making assumption that both the hands moves at constant angular velocities, the answer is ## \sqrt{7} .## But don't you think this assumption is somewhat doubtful and wrong?

Similar threads

  • · Replies 11 ·
Replies
11
Views
2K
  • · Replies 1 ·
Replies
1
Views
2K
  • · Replies 12 ·
Replies
12
Views
1K
  • · Replies 14 ·
Replies
14
Views
2K
  • · Replies 4 ·
Replies
4
Views
1K
Replies
4
Views
2K
  • · Replies 1 ·
Replies
1
Views
2K
  • · Replies 3 ·
Replies
3
Views
1K
  • · Replies 2 ·
Replies
2
Views
2K
  • · Replies 3 ·
Replies
3
Views
1K